Math questions from any Manhattan Prep GMAT Computer Adaptive Test.
Anne1276
 
 

When the positive integer x is divided by 4

by Anne1276 Sun Jun 17, 2007 2:50 pm

For (1) I understand why x must be an odd number but what information leads us to the fact that it must be a multiple of 3?


THANKS!
GMATPaduan
 
 

Appropriate Format

by GMATPaduan Mon Jun 18, 2007 10:46 am

Anne,

It would be helpful if you posted the entire question, so that everyone on the forum could benefit from the advice given by the instructors.

I also do not know if the instructors will answer a question, where the entire question is not writtten out and the source cited - they don't always have a OG book in front of them at all times.
Anne1276
 
 

by Anne1276 Mon Jun 18, 2007 1:26 pm

Here is the original problem:

When the positive integer x is divided by 4, is the remainder equal to 3?

(1) When x/3 is divided by 2, the remainder is 1.

(2) x is divisible by 5.
StaceyKoprince
ManhattanGMAT Staff
 
Posts: 9355
Joined: Wed Oct 19, 2005 9:05 am
Location: Montreal
 

by StaceyKoprince Tue Jun 19, 2007 2:41 am

Yes, we do need the entire question (and answer choices) as well as the source before we can answer the question. Anne, you list the source as CAT#1. Is this an MGMAT CAT or GMATPrep or some other source?
Stacey Koprince
Instructor
Director, Content & Curriculum
ManhattanPrep
Anne1276
 
 

MGMAT CAT #1

by Anne1276 Tue Jun 19, 2007 7:13 am

Sorry, it's my first time posting.

This is MGMAT CAT #1.
StaceyKoprince
ManhattanGMAT Staff
 
Posts: 9355
Joined: Wed Oct 19, 2005 9:05 am
Location: Montreal
 

MGMAT CAT DS problem

by StaceyKoprince Thu Jun 21, 2007 4:55 pm

No problem! If you look at the top of the list of threads in each forum, there's a sticky explaining how to post - take a look when you have a minute.

Statement 1 is a little tricky - it's got lots of info layered in there.

The statement tells us that when the combo (x/3) is divided by 2, the remainder is 1. Since the remainder is an integer, the number (x/3) itself must also be an integer. In order for (x/3) to be an integer, x must be divisible by 3. Otherwise, we would have a fraction or decimal amount for x/3.

For example 9 is divisible by 3, but 10 and 11 are not. (9/3) = 3 and 3/2 = 1 r.1. 10/3 = 3.333 so I can't get an integer remainder when dividing by 2.

Hope this helps!
Stacey Koprince
Instructor
Director, Content & Curriculum
ManhattanPrep
unique
 
 

Re: MGMAT CAT DS problem

by unique Tue Aug 21, 2007 2:41 pm

Can someone solve this using Algebra please. I made a mistake interpreting 1.

I treated x/3 divided by 2 as x/6 -- how can I avoid such mistakes?
StaceyKoprince
ManhattanGMAT Staff
 
Posts: 9355
Joined: Wed Oct 19, 2005 9:05 am
Location: Montreal
 

by StaceyKoprince Tue Aug 21, 2007 7:09 pm

You just need to study how to do this math so you know the problem is a remainder problem and you know how best to approach remainder problems. You could technically approach this with algebra, but that would make it an even harder problem than it already is.

Technically, if I say "when x/3 is divided by 2" ultimately I really am taking x and dividing it by 6 - so there isn't an error there to call it x/6. You just have to know how to continue to handle the information.

If I write it that way, though, I may not notice the key piece of info that x is divisible by 3 - so I just have to be careful. I should also notice that, if x has a remainder of 1 when I divide it by 2, then it must be odd.

So, I know from (1) that x is divisible by 3 and it is odd. So x could be, say, 3 or 9, but it can't be 6. If x = 9, the remainder is 1 when divided by 4. If x = 15 (the next largest possibility), the remainder is 3 when divided by 4. So I can't answer the question definitively yes or no. Insufficient.

(2) x is div. by 5. So x could be 5, 10, 15, etc. If x=5, the remainder is 1 when divided by 4. If x=15, the remainder is 3 when divided by 4. Once again, I can't answer definitively - insufficient.

(1) AND (2). Must be a multiple of 5 and 3, and be odd. Possibilities: 15, 45, 75, etc. I already know from previous work that the remainder is 3 when divided by 4. Let's try another. When 45 is divided by 4, the remainder is 1. I have "yes" and "no" again, so insufficient.

Answer is E.
Stacey Koprince
Instructor
Director, Content & Curriculum
ManhattanPrep
DCE
 
 

by DCE Sat Jul 26, 2008 5:00 am

Now Let's see how do we represent the first equation in Algebra

Whenever you are dividing consequently, pick the second half first i.e dividing by 2 leaves a remainder 1
This gives us: 2k+1

Now this has to be divisible by 3, so mutliply it by 3. Now we get: 3(2k +1). This is what it boils down to 6k +3.

Hope that this helps.

Thanks,
DCE
RonPurewal
Students
 
Posts: 19744
Joined: Tue Aug 14, 2007 8:23 am
 

by RonPurewal Thu Sep 04, 2008 7:16 am

DCE Wrote:Now Let's see how do we represent the first equation in Algebra

Whenever you are dividing consequently, pick the second half first i.e dividing by 2 leaves a remainder 1
This gives us: 2k+1

Now this has to be divisible by 3, so mutliply it by 3. Now we get: 3(2k +1). This is what it boils down to 6k +3.

Hope that this helps.

Thanks,
DCE


correct.

it may be more straightforward if you actually include x/3 in this calculation, because the problem is written with reference to the quantity x/3.

therefore:
"if you divide x/3 by 2, the remainder is 1"
--> x/3 = 2k + 1
multiply by 3 --> x = 6k + 3